Last visit was: 27 Apr 2024, 09:37 It is currently 27 Apr 2024, 09:37

Close

GRE Prep Club Daily Prep

Thank you for using the timer - this advanced tool can estimate your performance and suggest more practice questions. We have subscribed you to Daily Prep Questions via email.

Customized
for You

we will pick new questions that match your level based on your Timer History

Track
Your Progress

every week, we’ll send you an estimated GRE score based on your performance

Practice
Pays

we will pick new questions that match your level based on your Timer History

Not interested in getting valuable practice questions and articles delivered to your email? No problem, unsubscribe here.

Close

Request Expert Reply

Confirm Cancel
SORT BY:
Date
Verbal Expert
Joined: 18 Apr 2015
Posts: 28658
Own Kudos [?]: 33140 [0]
Given Kudos: 25178
Send PM
avatar
Intern
Intern
Joined: 15 Jun 2018
Posts: 5
Own Kudos [?]: 1 [0]
Given Kudos: 0
Send PM
Verbal Expert
Joined: 18 Apr 2015
Posts: 28658
Own Kudos [?]: 33140 [0]
Given Kudos: 25178
Send PM
avatar
Intern
Intern
Joined: 09 Aug 2018
Posts: 4
Own Kudos [?]: 1 [0]
Given Kudos: 0
Send PM
Re: The anticipated retirement of tens of thousands of baby boom [#permalink]
Why not C ??
Verbal Expert
Joined: 18 Apr 2015
Posts: 28658
Own Kudos [?]: 33140 [0]
Given Kudos: 25178
Send PM
Re: The anticipated retirement of tens of thousands of baby boom [#permalink]
Expert Reply
it is wrong because we care about of the Gov efforts not the skills for the best positon.

Hope this helps.

Regards
avatar
Intern
Intern
Joined: 23 Aug 2018
Posts: 8
Own Kudos [?]: 0 [0]
Given Kudos: 0
Send PM
Re: The anticipated retirement of tens of thousands of baby boom [#permalink]
I am asking for a strategy tip:
will the weaken question be addressed by the last sentence of the paragraph?
Verbal Expert
Joined: 18 Apr 2015
Posts: 28658
Own Kudos [?]: 33140 [0]
Given Kudos: 25178
Send PM
Re: The anticipated retirement of tens of thousands of baby boom [#permalink]
Expert Reply
Actually, the answer to your question is yes, even though you have to weaken the entire stem.

See my pdf. I hope it helps you.
Attachments

Critical reasoning strategies.pdf [943.05 KiB]
Downloaded 521 times

avatar
Intern
Intern
Joined: 24 Sep 2018
Posts: 32
Own Kudos [?]: 91 [0]
Given Kudos: 0
Send PM
Re: The anticipated retirement of tens of thousands of baby boom [#permalink]
1
The argument is that "To achieve this result (out of work youth filling roles), the city government should convene employers and educators to determine how best to create paths of upward mobility in these fields." A. is incorrect because it doesn't change the argument at all. C is also unrelated as the argument isn't specific to the top jobs. D affects nothing similar to answer A. E, while this would affect the results wanted (out of work youth filling roles), it has no bearing on the actual argument that the city gov should determine paths of upward mobility. B, on the other hand, that government efforts are ineffective, would mean that "the city government" doing something might not be the best way to achieve the results (out of work youth filling roles). The answer is B.
Prep Club for GRE Bot
[#permalink]
Moderators:
GRE Instructor
218 posts
GRE Instructor
1029 posts

Powered by phpBB © phpBB Group | Emoji artwork provided by EmojiOne